Đến nội dung

Hình ảnh

Thảo luận về Đề thi và Lời giải của IMO 2016

imo

  • Please log in to reply
Chủ đề này có 31 trả lời

#1
baopbc

baopbc

    Himura Kenshin

  • Thành viên nổi bật 2016
  • 410 Bài viết

*
Phổ biến

IMO.png

 

IMO 2016

 

Ngày 11-07-2016

 

 

Bài 1. Tam giác $BCF$ vuông tại $B.A$ là một điểm trên đường thẳng $CF$ sao cho $FA=FB,F$ nằm giữa $A$ và $C$. Chọn điểm $D$ sao cho $DA=DC$ và $AC$ là phân giác của $\angle DAB$. Chọn điểm $E$ sao cho $EA=ED$ và $AD$ là phân giác của $\angle EAC$. $M$ là trung điểm $CF$. $X$ là điểm thỏa mãn $AMXE$ là hình bình hành. Chứng minh rằng $BD$, $FX$ và $ME$ đồng quy.

 

Bài 2. Tìm tất cả các số tự nhiên $n$ sao cho có thể điền vào bảng ô vuông $n\times n$ các chữ cái $I$, $M$ và $O$ theo quy tắc

- Mỗi hàng và mỗi cột $I$, $M$, $O$ đều chiếm một phần ba số ô được điền.

- Trong bất kì đường chéo nào, nếu số ô được điền là bội của ba thì một phần ba trong số đó là $I$, một phần ba là $M$ và một phần ba là $O$.

 

Ghi chú. Các hàng và các cột của bảng $n\times n$ được đánh số từ $1$ đến $n$ theo thứ tự thông thường. Do đó mỗi ô đều tương ứng với một cặp số tự nhiên $(i,j)$ với $1\le i,j\le n$. Với $n>1$, bảng có $4n-2$ đường chéo được chia làm hai loại. Đường chéo loại 1 gồm các ô $(i,j)$ mà $i+j$ là hằng số và đường chéo loại 2 gồm các ô $(i,j)$ mà $i-j$ là hằng số.

 

Bài 3. Cho $P=A_1A_2\ldots A_k$ là một đa giác lồi trong mặt phẳng. Các đỉnh $A_1,A_2,\ldots A_k$ có tọa độ là các số nguyên và nằm trên một đường tròn. Gọi $S$ là diện tích của $P$. Một số tự nhiên $n$ lẻ thỏa mãn bình phương độ dài các cạnh của $P$ đều chia hết cho $n$. Chứng minh rằng $2S$ là một số tự nhiên chia hết cho $n$

 

Bài 4. Một tập hợp  các số nguyên dương được gọi là tập hương nếu tập hợp đó có ít nhất 2 phần tử và mỗi phần tử của nó đều có ước nguyên tố chung với ít nhất một trong các phần tử còn lại . Đặt $P(n)=n^{2}+n+1$. Hãy tìm số nguyên dương $b$ nhỏ nhất sao cho tồn tại số không âm $a$  để tập hợp  $\left \{ P(a+1);P(a+2);...;P(a+b) \right \}$ là tập hương.

 

Bài 5. Người ta viết lên bảng phương trình:

$(x-1)(x-2)(x-3)...(x-2016)=(x-1)(x-2)(x-3)...(x-2016)$

với 2016 nhân tử bậc nhất ở mỗi vế. Hãy tìm số nguyên dương $k$ nhỏ nhất để có thể xóa đi $k$ nhân tử trong số 4032 nhân tử nêu trên sao cho mỗi vế còn ít nhất một nhân tử và phương trình thu được không có nghiệm thực.

 

Bài 6. Trong mặt phẳng, cho $n\geq 2$ đoạn thẳng sao cho 2 đoạn thẳng bất kì cắt nhau tại một điểm nằm trên mỗi đoạn và không có ba đoạn thẳng nào đồng quy.Với mỗi đoạn thẳng thầy Minh chọn một đầu mút của nó rồi đặt lên đó một con ếch sao cho mặt con ếch hướng về đầu mút còn lại. Sau đó thầy vỗ tay $n-1$ lần. Mỗi lần vỗ tay con ếch ngay lập tức nhảy đến giao điểm gần nhất trên đoạn thẳng của nó. Tất cả những con ếch đều không thay đổi  hướng nhảy của mình trong toàn bộ quá trình nhảy. Thầy Minh muốn đặt các con ếch sao cho sau mỗi lần vỗ tay không có hai con nào nhảy đến cùng một điểm.

(a). Chứng minh rằng thầy Minh luôn thực hiện được ý định của mình nếu $n$ là số lẻ.

(b).  Chứng minh rằng thầy Minh không thể thực hiện được ý định của mình nếu nếu $n$ là số chẵn.


Bài viết đã được chỉnh sửa nội dung bởi baopbc: 12-07-2016 - 17:03
Update ngày 2


#2
tranquocluat_ht

tranquocluat_ht

    Thượng sĩ

  • Thành viên
  • 235 Bài viết
 

Ngày 11-07-2016

 

 

Bài 1. Tam giác $BCF$ vuông tại $B.A$ là một điểm trên đường thẳng $CF$ sao cho $FA=FB,F$ nằm giữa $A$ và $C$. Chọn điểm $D$ sao cho $DA=DC$ và $AC$ là phân giác của $\angle DAB$. Chọn điểm $E$ sao cho $EA=ED$ và $AD$ là phân giác của $\angle EAC$. $M$ là trung điểm $CF$. $X$ là điểm thỏa mãn $AMXE$ là hình bình hành. Chứng minh rằng $BD$, $FX$ và $ME$ đồng quy.

 

 

756.jpg?dl=0

Mấu chốt là chứng minh $B, F, E$ thẳng hàng.

Giả sử vị trí các điểm như hình vẽ, các trường hợp khác chứng minh tương tự.

Thật vậy, sử dụng kỹ năng gọi điểm phẩy. Giả sử $BF$ cắt đường thẳng qua $D$ song song với $AC$ tại $E'.$

Ta sẽ chứng minh $E'$ trùng $E.$

Ta có $\widehat{E'DA}=\widehat{DAC}=\widehat{E'BA}$ nên tứ giác $E'DBA$ nội tiếp. 

Gọi $(E'DBA)$ cắt $AC$ tại $M'$ khác $A.$ Ta sẽ chứng minh $M'$ trùng $M$.

Thật vậy, ta có $\widehat{DM'B}=180^0-\widehat{DE'B}=180^0-2 \widehat{CAB}=2 \widehat{DCB}$ (chú ý $DE'M'C$ là hình bình hành vì có 2 cặp cạnh đối song song nên $\widehat{M'CD}=\widehat{M'E'D}=\widehat{CAB}$) nên $M'$ là tâm nội tiếp tam giác $DCB.$

Cũng do $DE'M'C$ là hình bình hành nên $DE'=CM'=M'D$ suy ra luôn $M'$ trùng $M$ và $E'$ trùng $E.$ 

Sau khi có $B, F, E$ thằng hàng thì $EX=MA=EB, ED=FM=EF$ và $MX=AE=MB$ từ đó có đpcm.


Bài viết đã được chỉnh sửa nội dung bởi tranquocluat_ht: 11-07-2016 - 18:05


#3
Bui Ba Anh

Bui Ba Anh

    Thiếu úy

  • Thành viên
  • 562 Bài viết

Bài $1$ khá hay. Gọi $CD$ cắt $BF$ tại $T$ và dễ dàng chứng minh $E$ là trung điểm $FT$ (Chú ý cách xác định điểm $E$ là giao của trung trực $AD$ và đường thẳng đổi xứng của $AC$ qua $AD$)

Khi đó tứ giác $XMFE$ nội tiếp, ngũ giác $BMDEA$ và $BFDXC$ nội tiếp.

Suy ra $EM,FX,BD$ đồng quy tại tâm đẳng phương của ba đường tròn.


Bài viết đã được chỉnh sửa nội dung bởi baopbc: 11-07-2016 - 15:45

NgọaLong

#4
perfectstrong

perfectstrong

    $LOVE(x)|_{x =\alpha}^\Omega=+\infty$

  • Quản lý Toán Ứng dụng
  • 4991 Bài viết

Không biết có phải trùng hợp hay không nhưng bài 2 có cấu hình "khá giống" với bài 4 VMO 2016 :D

 

http://diendantoanho...-giải-vmo-2016/


Luôn yêu để sống, luôn sống để học toán, luôn học toán để yêu!!! :D
$$\text{LOVE}\left( x \right)|_{x = \alpha}^\Omega = + \infty $$
I'm still there everywhere.

#5
QQspeed22

QQspeed22

    Hạ sĩ

  • Thành viên
  • 73 Bài viết

Bài $1$ khá hay. Gọi $CD$ cắt $BF$ tại $T$ và dễ dàng chứng minh $E$ là trung điểm $FT$ (Chú ý cách xác định điểm $E$ là giao của trung trực $AD$ và đường thẳng đổi xứng của $AC$ qua $AD$)

Khi đó tứ giác $XMFE$ nội tiếp, ngũ giác $BMDEA$ và $BFDXC$ nội tiếp.

Suy ra $EM,FX,BD$ đồng quy tại tâm đẳng phương của ba đường tròn.

Sao chứng minh được E là trung điểm FT bạn



#6
QQspeed22

QQspeed22

    Hạ sĩ

  • Thành viên
  • 73 Bài viết

Lí do $\widehat{DAC}=\widehat{E'BA}$

@Ego: Nhắc nhở bạn QQspeed tránh spam các bài viết và đừng quote cả một lời giải, trông khó nhìn lắm



#7
quanghung86

quanghung86

    Thiếu úy

  • Điều hành viên
  • 632 Bài viết

Bài hình P1 khá thú vị. Mình xin gửi lại hai mở rộng cho bài 1 mình đã post trên AoPS như sau

 

Bài toán 1a. Cho tam giác $ABC$ có $\angle ABC$ tù và tâm ngoại tiếp $D$. Trung trực $AB$ cắt $AC$ tại $F$. $E,M$ là tâm ngoại tiếp tam giác $ADF,BFC$. Dựng hình bình hành $AMXE$. Chứng minh rằng $EM,FX,BD$ đồng quy.

 

Figure3959.png

 

Bài toán 1b. Cho tam giác $ABC$ tâm ngoại tiếp $O$. Trung trực của $AB,AC$ cắt $BC$ tại $F,E$. Gọi $K,L,M,N$ là tâm ngoại tiếp các tam giác $BFD,CDE,AFC,AEB$.

 

a) Chứng minh rằng $KM,LN,AO$ đồng quy.

 

b) Chứng minh rằng $KL$ vuông góc với đường thẳng Euler của tam giác $ABC$.

 

Figure3960.png



#8
QQspeed22

QQspeed22

    Hạ sĩ

  • Thành viên
  • 73 Bài viết

Các bạn giải câu 1 mình xem có gì đó thiếu tự nhiên thì phải



#9
tranquocluat_ht

tranquocluat_ht

    Thượng sĩ

  • Thành viên
  • 235 Bài viết

Lí do $\widehat{DAC}=\widehat{E'BA}$

Chỗ đó hiển nhiên vì cùng bằng $\widehat{FBA}$.



#10
QQspeed22

QQspeed22

    Hạ sĩ

  • Thành viên
  • 73 Bài viết

Chỗ đó hiển nhiên vì cùng bằng $\widehat{FBA}$.

Đã nội tiếp đâu sao bằng nhau dc



#11
tranquocluat_ht

tranquocluat_ht

    Thượng sĩ

  • Thành viên
  • 235 Bài viết

Đã nội tiếp đâu sao bằng nhau dc

Bằng nhau vì lý do khác chứ không phải vì nội tiếp.



#12
Hoang Nhat Tuan

Hoang Nhat Tuan

    Hỏa Long

  • Thành viên
  • 974 Bài viết

Bài hình P1 khá thú vị. Mình xin gửi lại hai mở rộng cho bài 1 mình đã post trên AoPS như sau

 

Bài toán 1a. Cho tam giác $ABC$ có $\angle ABC$ tù và tâm ngoại tiếp $D$. Trung trực $AB$ cắt $AC$ tại $F$. $E,M$ là tâm ngoại tiếp tam giác $ADF,BFC$. Dựng hình bình hành $AMXE$. Chứng minh rằng $EM,FX,BD$ đồng quy.

 

attachicon.gifFigure3959.png

 

Bài toán 1b. Cho tam giác $ABC$ tâm ngoại tiếp $O$. Trung trực của $AB,AC$ cắt $BC$ tại $F,E$. Gọi $K,L,M,N$ là tâm ngoại tiếp các tam giác $BFD,CDE,AFC,AEB$.

 

a) Chứng minh rằng $KM,LN,AO$ đồng quy.

 

b) Chứng minh rằng $KL$ vuông góc với đường thẳng Euler của tam giác $ABC$.

 

attachicon.gifFigure3960.png

Em xin giải mở rộng của thầy

Bài toán $1a$: Ta có: $\widehat{BDF}=\frac{1}{2}\widehat{BDA}=\widehat{BCA}$ nên tứ giác $BCDF$ nội tiếp đường tròn tâm $M$

Hai tam giác $ADF$ và $BDF$ đối xứng nhau qua $DF$ nên $E$ đối xứng $M$ qua $DF$

Lại có $MF=MD$ nên $ME$ là đường trung trực của $FD$. Do đó phép đối xứng qua trung điểm của $DF$ biến $A$ thành $X$ hay $ADXF$ là hình bình hành.

$\Rightarrow \widehat{FXD}=\widehat{FAD}=\widehat{FBD}$ hay $BFDX$ nội tiếp đường tròn $(M)$, cũng dễ dàng chứng minh được $FD|| BX$ nên $FDXB$ là hình thang cân.

$ME$ là đường trung trực của $FD$ nên hiển nhiên $ME$ đi qua giao điểm của $XF$ và $BD$

Câu $b$ của thầy chắc là $BFO,COE$ thay vì $BFD,CDE$ chứ ạ

Theo câu $a$ thì $KM,LN$ lần lượt là trung trực của $OF,OE$, do đó để chứng minh $KM,LN,AO$ đồng quy thì ta cần chứng minh $AO$ đi qua tâm đường tròn ngoại tiếp tam giác $OEF$

Ta có $\widehat{AOF}=\widehat{C}=\widehat{EOH}$ với $H$ là chân đường vuông góc hạ từ $O$ xuống $BC$ nên $OA, OH$ đẳng giác

Suy ra đpcm

Spoiler
 

Ý $b$ chỉ cần biến đổi góc là ra: $BH\cap (K)=X, CH\cap (L)=Y$ ta có:

$\widehat{XOY}=\widehat{XOF}+\widehat{FOE}+\widehat{EOY}=\widehat{HBC}+\widehat{BHC}+\widehat{HCB}=180\circ$ nên $X,O,Y$ thẳng hàng.

Tiếp tục xét tứ giác $XBYC$ thì dễ dàng chứng minh được $\widehat{BXY}=\widehat{YCB}$ nên tứ giác $XBYC$ nội tiếp

Do đó $HB.HX=HC.HY$ nên $H$ thuộc trục đẳng phương của 2 đường tròn $(K),(L)$.

Suy ra $HO\perp KL$ 


Bài viết đã được chỉnh sửa nội dung bởi Hoang Nhat Tuan: 11-07-2016 - 22:45

Ngài có thể trói cơ thể tôi, buộc tay tôi, điều khiển hành động của tôi: ngài mạnh nhất, và xã hội cho ngài thêm quyền lực; nhưng với ý chí của tôi, thưa ngài, ngài không thể làm gì được.

#13
JUV

JUV

    Trung sĩ

  • Điều hành viên OLYMPIC
  • 138 Bài viết

Bài 2: Dễ thấy $n\vdots 3$, đặt $n=3t$, chia bảng $n\times n$ thành $t^2$ bảng con $3\times 3$ thì lúc này xét $t$ hàng và $t$ cột sao cho mỗi hàng và cột đó đều đi qua ô chính giữa của $t$ bảng $3\times 3$, ta cũng xét $4n-2$ đường chéo như đề bài cho, thấy rằng mỗi đường chéo này đều đi qua ô chính giữa của $1$ số bảng $3\times 3$ .Gọi $t$ hàng, $t$ cột và $4n-2$ đường chéo đã xét là các đường, mỗi ô chính giữa của các bảng $3\times 3$ đều có đúng $3$ đường đi qua và các ô khác chỉ có đúng $1$ đường đi qua. Mà theo đầu bài thì mỗi đường chứa các chữ $I,M,O$ bằng nhau và cũng dễ nhận thấy rằng số các chữ cái $I,M,O$ viết trên bảng cũng bằng nhau nên nếu xét $t^2$ ô là ô chính giữa của $t^2$ bảng thì số các ô được viết chữ $I,M,O$ là bằng nhau. Vì vậy $t^2\vdots 3$ nên $n\vdots 9$. Xét trường hợp $n=9$ có thể dễ dàng chỉ ra $1$ bảng thoả mãn. Còn với $n=9k$ với $k\in \mathbb{N}$ bất kì thì chỉ cần ghép $k^2$ bảng $9\times 9$ thoả mãn đề bài với nhau để tạo được $1$ bảng $9k\times 9k$ thoả mãn đề bài



#14
JUV

JUV

    Trung sĩ

  • Điều hành viên OLYMPIC
  • 138 Bài viết

Có vẻ tình hình đội tuyển VN không ổn rồi. Tổ hợp là điểm yếu của đoàn mà ngay ngày đầu tiên đã phang 2 câu khó chơi rồi



#15
lenhatsinh3

lenhatsinh3

    Hạ sĩ

  • Thành viên
  • 86 Bài viết

 

attachicon.gifIMO.png

 

IMO 2016

 

Ngày 11-07-2016

 

 

Bài 1. Tam giác $BCF$ vuông tại $B.A$ là một điểm trên đường thẳng $CF$ sao cho $FA=FB,F$ nằm giữa $A$ và $C$. Chọn điểm $D$ sao cho $DA=DC$ và $AC$ là phân giác của $\angle DAB$. Chọn điểm $E$ sao cho $EA=ED$ và $AD$ là phân giác của $\angle EAC$. $M$ là trung điểm $CF$. $X$ là điểm thỏa mãn $AMXE$ là hình bình hành. Chứng minh rằng $BD$, $FX$ và $ME$ đồng quy.

 

 

 

 

12.png


Bài viết đã được chỉnh sửa nội dung bởi lenhatsinh3: 12-07-2016 - 13:07

:ukliam2:  :ukliam2:  :ukliam2:  :ukliam2:  :ukliam2:

      :ukliam2:

            :ukliam2:

                  :ukliam2:

             :ukliam2:

        :ukliam2:  

     :ukliam2:  :ukliam2:  :ukliam2:  :ukliam2:  :ukliam2:


#16
JUV

JUV

    Trung sĩ

  • Điều hành viên OLYMPIC
  • 138 Bài viết

Không biết có phải trùng hợp hay không nhưng bài 2 có cấu hình "khá giống" với bài 4 VMO 2016 :D

 

http://diendantoanho...-giải-vmo-2016/

Mình có thấy trùng tí nào đâu. Bài VMO thì lời giải trâu bò hơn nhưng bài IMO mới là bài khó hơn



#17
perfectstrong

perfectstrong

    $LOVE(x)|_{x =\alpha}^\Omega=+\infty$

  • Quản lý Toán Ứng dụng
  • 4991 Bài viết

Mình có thấy trùng tí nào đâu. Bài VMO thì lời giải trâu bò hơn nhưng bài IMO mới là bài khó hơn

Mình đâu có nói gì tới "trùng". Nếu mà "trùng" thì mới lớn chuyện. Còn đây chỉ là nhận xét về "thể loại, kết cấu" thôi.


Luôn yêu để sống, luôn sống để học toán, luôn học toán để yêu!!! :D
$$\text{LOVE}\left( x \right)|_{x = \alpha}^\Omega = + \infty $$
I'm still there everywhere.

#18
moonkey01

moonkey01

    Hạ sĩ

  • Thành viên
  • 50 Bài viết

Bài 5: Đẳng thức $(x-1)(x-2)...(x-2016)=(x-1)(x-2)...(x-2016)$ được viết trên bảng với 2016 thừa số ở mỗi vế. Xác định giá trị nhỏ nhất có thể của $k$ để ta có thể xoá $k$ trong số 4032 thừa số trên, với điều kiện là mỗi vế còn lại ít nhất một thừa số, để phương trình nhận được sau phép biến đổi là vô nghiệm.

 

Giải: Ta sẽ chứng minh rằng $k=2016$ là giá trị nhỏ nhất cần tìm.

 

Nếu như ta xoá ít hơn 2016 thừa số, thì đẳng thức nhận được chứa ít nhất 2017 thừa số. Do có 2016 loại thừa số là $x-1$, $x-2$,..., $x-2016$ nên theo nguyên lý Dirichlet, tồn tại một thừa số xuất hiện 2 lần. Thừa số ấy xuất hiện ở cả 2 vế của đẳng thức, dẫn đến việc phương trình nhận được sẽ có nghiệm. Do đó $k\geqslant 2016$

 

Xét phương trình $(x-1)(x-3)...(x-2015)=(x-2)(x-4)...(x-2016)$ $(1)$ nhận được sau khi xoá $k=2016$ thừa số từ đẳng thức ban đầu.

 

Trường hợp $x\in \left \{ 1;2;...;2016 \right \}$ thì phương trình hiển nhiên vô nghiệm.

 

Với $x< 1$ thì $2015-x<2016-x$ ,..., $1-x<2-x$. Cả 2 vế của các bất đẳng thức trên đều dương nên nhân vế theo vế, $VT(1)<VP(1)$. Phương trình vô nghiệm.

 

Với $1<x<2$ thì $VT(1)<0<VP(1)$ nên phương trình cũng vô nghiệm.

 

Với $2<x<3$ thì $2015-x<2016-x$ ,..., $3-x<4-x$, $x-1<x-2$. Cả 2 vế các BĐT trên đều dương nên nhân vế theo vế và đổi dấu, $VT(1)>VP(1)$. Phương trình vô nghiệm.

 

Với $3<x<4$ thì $VT(1)>0>VP(1)$ nên phương trình cũng vô nghiệm.

 

Các trường hợp còn lại ta chứng minh tương tự.

 

Kết luận: Giá trị nhỏ nhất của k là 2016.


Bài viết đã được chỉnh sửa nội dung bởi moonkey01: 12-07-2016 - 13:58


#19
IHateMath

IHateMath

    Thượng sĩ

  • Thành viên
  • 299 Bài viết

File gửi kèm  2016-vie.pdf   168.36K   757 Số lần tải

Đề phiên bản Tiếng Việt đây, các mem chém đi nhé :D

 



#20
the unknown

the unknown

    Thượng sĩ

  • Thành viên
  • 208 Bài viết

 

attachicon.gifIMO.png

 

 

Bài 3. Cho $P=A_1A_2\ldots A_k$ là một đa giác lồi trong mặt phẳng. Các đỉnh $A_1,A_2,\ldots A_k$ có tọa độ là các số nguyên và nằm trên một đường tròn. Gọi $S$ là diện tích của $P$. Một số tự nhiên $n$ lẻ thỏa mãn bình phương độ dài các cạnh của $P$ đều chia hết cho $n$. Chứng minh rằng $2S$ là một số tự nhiên chia hết cho $n$

 

Đây là một lời giải em tham khảo ở AoPS cho bài số 3, bài được cho là khó nhất của IMO năm nay  :)

Bài 3: Trước hết, giả sử rằng đa giác $A_1A_2\ldots A_k$ có một đường chéo thỏa mãn bình phương của đường chéo đó là một số nguyên chia hết cho $n$, khi đó ta sẽ cắt đa giác này bằng đường chéo này thành hai đa giác với số cạnh nhỏ hơn. Do đó không giảm tổng quát có thể giả sử đa giác $A_1A_2\ldots A_k$ không có một đường chéo nào có bình phương chia hết cho $n$.

 

Giả sử $k\ge 4$. Không mất tính tổng quát, ta sẽ chứng minh cho trường hợp $n=p^a$ ($p$ là số nguyên tố lẻ). Khi đó, xét một số tự nhiên $r$ thỏa $a>r\geq 0$ và $p^r\mid A_iA_j^2$ với mọi $i\neq j$, $0<i,j\le k$. Hiển nhiên ta có $r+1\leq a$ nên $p^{r+1}\mid p^a\mid A_iA_j^2$ nếu $A_i,A_j$ là hai đỉnh kề nhau.

 

Ta sẽ chứng minh bằng quy nạp rằng nếu $r$ là một số tự nhiên thỏa mãn điều kiện trên thì $r+1$ cũng sẽ thỏa mãn các điều kiện trên. Do tất cả các đỉnh của đa giác cùng nằm trên một đường tròn nên xét cụ thể cho một tứ giác $A_{i-1}A_iA_{i+1}A_j$ và cho đơn giản, ta đặt: $A_{i-1}A_i=b,A_{i}A_{i+1}=c,A_{i+1}A_j=d,A_jA_{i-1}=e,A_{i-1}A_{i+1}=f,A_iA_j=g$.

 

Và theo giả thiết quy nạp, ta đặt: $b^2=p^ax,c^2=p^ay,d^2=p^rz,e^2=p^rk,f^2=p^rw,g^2=p^rt$ ( với $x,y,z,k,w,t$ là các số nguyên) Khi đó theo định lý Ptolemy thì ta có được:$bd+ce=fg \Rightarrow f^2g^2=b^2d^2+c^2e^2+2bcde=p^{a+r}(xz+yk+2\sqrt{xyzk})$. Từ đó ta có $\sqrt{xyzk}$ là một số hữu tỉ nên cũng là một số nguyên. Do đó ta có được: $p^{a+r}\mid f^2g^2\Rightarrow v_{p}(f^2g^2)\geq a+r$. Tuy nhiên: vì $A_iA_j$ và $A_{i-1}A_{i+1}$ là các đường chéo nên $v_{p}(f^2)\leq a-1\Rightarrow v_{p}(g^2)\geq r+1\Rightarrow p^{r+1}\mid g^2$

 

Tức là khi đó $p^{r+1}\mid A_iA_j^2$ với $A_iA_j$ là một đường chéo và tương tự với các đường chéo khác, ta có được $p^{r+1}\mid A_iA_j^2$ với mọi $i\neq j$, $0<i,j\leq k$. Như vậy theo nguyên lí quy nạp, giả thiết được chứng minh. Khi đó theo giả thiết thì $p^{a-1}\mid A_iA_j^2$ với mọi $i\neq j$ và cũng chứng minh tương tự mà $p^a\mid A_iA_j^2$ với mọi $i\neq j$. Đây là điều vô lí ( do ta đã giả sử rằng không có đường chéo nào chia hết cho $p^a$).

 

Do đó $k<4$, tức là ta sẽ quy về trường hợp đơn giản $k=3$. Ta có thể chứng minh điều này cách dễ dàng vì các đỉnh $A_1,A_2,A_3$ có tọa độ nguyên nên dễ dàng chứng minh được $2S$ là số nguyên.

Hơn nữa theo công thức Hê-rông thì $2S=\frac{\sqrt{4a^2b^2-(a^2+b^2-c^2)^2}}{2}$ ($a,b,c$ là các cạnh của tam giác) nên $4.(2S)^2=(4a^2b^2-(a^2+b^2-c^2)^2)\vdots n^2$. Do $n$ lẻ nên $(4,n)=1$ do đó $n^2\mid (2S)^2$ nên $n\mid 2S$.

Vậy bài toán được chứng minh hoàn toàn.$\blacksquare$

 

P.s: Hình như ngày 2 không có hình  :(


Bài viết đã được chỉnh sửa nội dung bởi the unknown: 13-07-2016 - 20:45

$\texttt{If you don't know where you are going, any road will get you there}$






Được gắn nhãn với một hoặc nhiều trong số những từ khóa sau: imo

1 người đang xem chủ đề

0 thành viên, 1 khách, 0 thành viên ẩn danh